Đến nội dung

Nguyenhuyen_AG nội dung

Có 785 mục bởi Nguyenhuyen_AG (Tìm giới hạn từ 20-04-2020)



Sắp theo                Sắp xếp  

#585265 Topic về Bất đẳng thức, cực trị THCS

Đã gửi bởi Nguyenhuyen_AG on 26-08-2015 - 23:25 trong Bất đẳng thức và cực trị

$\frac{a^{2}+b^{2}}{a+b} +  \frac{b^{2}+c^{2}}{b+c} + \frac{c^{2}+a^{2}}{c+a}   \leq  \frac{3.(a^{2}+b^{2}+c^{2})}{a+b+c}$ 

 

\[\frac{(a+b+c)(a^2+b^2)}{a+b} = a^2+b^2+c(a+b)-\frac{2abc}{a+b}.\]

Nên bất đẳng thức cần chứng minh tương đương với

\[a^2+b^2+c^2+2abc\left(\frac{1}{a+b}+\frac{1}{b+c}+\frac{1}{c+a}\right) \geqslant 2(ab+bc+ca).\]

Hiển nhiên đúng theo bất đẳng thức Cauchy-Schwarz và Schur.




#588626 Topic về Bất đẳng thức, cực trị THCS

Đã gửi bởi Nguyenhuyen_AG on 12-09-2015 - 23:58 trong Bất đẳng thức và cực trị

Chứng minh : $\frac{a^{3} - b^{3}}{\left ( a - b \right )^{3}} + \frac{b^{3} - c^{3}}{\left ( b - c \right )^{3}} + \frac{c^{3} - a^{3}}{\left ( c - a \right )^{3}} \geq \frac{9}{4}$ với $a , b , c$ từng đôi khác nhau

 

Xét hiệu hai vế ta có \[\sum \frac{a^{3} - b^{3}}{\left ( a - b \right )^{3}} - \frac{9}{4} = \frac{3[ab(a+b)+bc(b+c)+ca(c+a)-6abc]^2}{4(a-b)^2(b-c)^2(c-a)^2} \geqslant 0.\] Đẳng thức xảy ra khi và chỉ khi

\[\left\{\begin{aligned} & ab(a+b)+bc(b+c)+ca(c+a) = 6abc \\ & (a-b)(b-c)(c-a) \ne 0 \end{aligned}\right.\] Bài toán được chứng minh.




#566074 Topic tổng hợp một số bất đẳng thức trong kì thi MO các nước

Đã gửi bởi Nguyenhuyen_AG on 15-06-2015 - 23:49 trong Bất đẳng thức - Cực trị

  Bài 72 (IMO 2005): Cho các số thực dương $a,b,c$ thỏa mãn $abc\geq 1$. CMR:

 

      $\frac{a^5-a^2}{a^5+b^2+c^2}+\frac{b^5-b^2}{b^5+c^2+a^2}+\frac{c^5-c^2}{c^5+a^2+b^2}\geq 0$

 

Lời giải đặc biệt của kỳ thi.

 

Ta có

\[\frac{a^5-a^2}{a^5+b^2+c^2}-\frac{a^5-a^2}{a^3(a^2+b^2+c^2)}=\frac{(a-1)^2(a^2+a+1)(b^2+c^2)}{a(a^5+b^2+c^2)(a^2+b^2+c^2)} \ge 0.\]

Suy ra

\[\sum \frac{a^5-a^2}{a^5+b^2+c^2} \ge \sum\frac{a^5-a^2}{a^3(a^2+b^2+c^2)} = \sum \frac{a^2-\frac{1}{a}}{a^2+b^2+c^2} \ge \sum \frac{\sum a^2-\sum bc}{a^2+b^2+c^2} \ge 0.\]
---------------

@ducvipdh12: chào mừng sự trở lại của anh Huyện,lâu lắm mới thấy anh onl diễn đàn :))




#567744 Topic tổng hợp một số bất đẳng thức trong kì thi MO các nước

Đã gửi bởi Nguyenhuyen_AG on 23-06-2015 - 22:07 trong Bất đẳng thức - Cực trị

Không biết bài này có chưa, nếu có rồi thì anh Đức xóa dùm em

Bài 142(APMO 1996) Cho a,b,c là độ dài 3 cạnh tam giác. Chứng minh rằng:

$\sqrt{a+b-c}+\sqrt{b+c-}+\sqrt{c+a-b}\leq \sqrt{a}+\sqrt{b}+\sqrt{c}$

Đẳng thức xảy ra khi nào

 

Áp dụng bất đẳng thức Cauchy-Schwarz ta có  \[\sqrt{a+b-c}+\sqrt{b+c-a} \leqslant \sqrt{2[(a+b-c)+(b+c-a)]} = 2\sqrt{b}.\] Đánh giá tương tự cho hai đại lượng còn lại ta có điều phải chứng minh.




#567808 Topic tổng hợp một số bất đẳng thức trong kì thi MO các nước

Đã gửi bởi Nguyenhuyen_AG on 24-06-2015 - 11:29 trong Bất đẳng thức - Cực trị

Bài 114 (VMO 1999) : Xét các số thực dương $a,b$ sao cho phương trình : 

                                                                                     $ax^{3}-x^{2}+bx-1=0$

có ba nghiệm thực dương ( các nghiệm có thể bằng nhau).

 

Tìm giá trị nhỏ nhất của biểu thức $P=\frac{5a^{2}-3ab+2}{a^{2}(b-a)}$ .

 

Gọi $u,s,v$ là ba nghiệm của phương trình đã cho, theo định lý Viète ta có

 

\[\left\{ \begin{aligned} & u+s+v=\frac{1}{a} \\ & us+sv+vu=\frac{b}{a} \\ & usv=\frac{1}{a}\end{aligned} \right.\]

Đặt $c=\frac{1}{a},$ áp dụng bất đẳng thức AM-GM, ta có

\[c = usv = u+s+v \geqslant 3\sqrt[3]{usv}=3\sqrt[3]{c},\]

suy ra $c \geqslant 3\sqrt{3}.$ Lại có

\[c^2 = (u+s+v)^2 \geqslant 3(us+sv+vu) = 3bc.\]

Từ những điều này ta có biến đổi

\[P = \frac{1}{a}\cdot\frac{5-\dfrac{3b}{a}+\dfrac{2}{a^2}}{\dfrac{b}{a}-1} \geqslant \frac{c(5-3bc+2c^2)}{bc-1} \geqslant \frac{c(5-c^2+2c^2)}{\dfrac{c^2}{3}-1} = \frac{3c(c^2+5)}{c^2-3}.\]

Tách

\[\frac{c(c^2+5)}{c^2-3} = c\left ( 1+\frac{8}{c^2-3} \right ) = \frac{c+\sqrt{3}}{4} + \left [\frac{c+\sqrt{3}}{4} + \frac{c-\sqrt{3}}{2} + \frac{8c}{(c-\sqrt{3})(c+\sqrt{3})}  \right ],\]

rồi áp dụng bất đẳng thức AM-GM ta thấy

\[\begin{aligned}\frac{c(c^2+5)}{c^2-3} & \geqslant \frac{c+\sqrt{3}}{4} + 3\sqrt[3]{\frac{c+\sqrt{3}}{4} \cdot \frac{c-\sqrt{3}}{2} \cdot \frac{8c}{(c-\sqrt{3})(c+\sqrt{3})}}  \\& = \frac{c+\sqrt{3}}{4}+3\sqrt[3]{c} \\& \geqslant \frac{3\sqrt{3}+\sqrt{3}}{4}+3\sqrt[3]{3\sqrt{3}} \\& = 4\sqrt{3}.\end{aligned}\]

Do đó $P \geqslant 3\cdot 4\sqrt{3} = 12\sqrt{3}.$ Ngoài ra nếu $a = \frac{1}{3\sqrt{3}},\,b=\sqrt{3}$ thì $P = 12\sqrt{3}.$ Điều này cho phép ta kết luận $P_{\min} = 12\sqrt{3}.$

 

Nhận xét. Sau khi đưa bài toán về tìm giá trị nhỏ nhất của hàm một biến $F = \frac{3c(c^2+5)}{c^2-3}$ với điều kiện $c \geqslant 3\sqrt{3}$ chúng ta có nhiều cách tiếp cận khác nhau như biến đổi tương đương hoặc đạo hàm, ... tuy nhiên anh thích cách dùng các bất đẳng thức cổ điển hơn nên chọn cách này.

 

Việc tách đẳng thức

\[\frac{c(c^2+5)}{c^2-3} = c\left ( 1+\frac{8}{c^2-3} \right ) = \frac{c+\sqrt{3}}{4} + \left [\frac{c+\sqrt{3}}{4} + \frac{c-\sqrt{3}}{2} + \frac{8c}{(c-\sqrt{3})(c+\sqrt{3})}  \right ],\]

là dự vào dự đoán đẳng thức xảy ra khi $c = 3\sqrt{3}$ chứ không có gì ảo diệu hay ma thuật nào ở đây cả. :D




#671111 Topic tổng hợp một số bất đẳng thức trong kì thi MO các nước

Đã gửi bởi Nguyenhuyen_AG on 11-02-2017 - 15:53 trong Bất đẳng thức - Cực trị

Phương pháp trội tử tức là giá trị bằng $1/2$ cộng thêm $1$ ẩn để thêm vào nữa.

Bài tập:

$(1)$

$\sum (\frac{a}{a+b})^2+3\geqslant \frac{5}{2}\sum \frac{a}{a+b}$

Vân vân :v

 

Ta có

\[\sum \frac{a^2}{(a+b)^2} +3 - \frac{5}{2}\sum \frac{a}{a+b} = \frac{1}{2(a+b)^2(b+c)^2(c+a)^2}\sum ca(ab+3bc+3ca+c^2)(a-b)^2 \geqslant 0.\]




#671109 Topic tổng hợp một số bất đẳng thức trong kì thi MO các nước

Đã gửi bởi Nguyenhuyen_AG on 11-02-2017 - 15:46 trong Bất đẳng thức - Cực trị

Cho ba số thực $a,b,c$ thỏa mãn $a^2+b^2+c^2>0.$ Chứng minh rằng:

$$\frac{(2a+b+c)^2}{2a^2+(b+c)^2}+\frac{(2b+c+a)^2}{2b^2+(c+a)^2}+\frac{(2c+a+b)^2}{2c^2+(a+b)^2}\leq 8.$$




#567162 Topic tổng hợp một số bất đẳng thức trong kì thi MO các nước

Đã gửi bởi Nguyenhuyen_AG on 20-06-2015 - 20:33 trong Bất đẳng thức - Cực trị

Bài 133. Cho $a,\,b,\,c$ là ba số thực dương bất kỳ và đặt $k=abc.$ chứng minh rằng

\[\frac{1}{(a+1)^3}+\frac{1}{(b+1)^3}+\frac{1}{(c+1)^3} \geqslant \frac{-k^3+3k^2+k+1}{2(k+1)^3}.\]

Nếu $k=1$ thì ta được bất đẳng thức trong đề thi Việt Nam TST 2005.




#671110 Topic tổng hợp một số bất đẳng thức trong kì thi MO các nước

Đã gửi bởi Nguyenhuyen_AG on 11-02-2017 - 15:49 trong Bất đẳng thức - Cực trị

\[\frac{a}{\sqrt{a+b}}+\frac{b}{\sqrt{b+c}}+\frac{c}{\sqrt{c+a}}\leq 4\sqrt{a+b+c}\]
 

Với mọi a,b,c ko âm,CM bđt trên đúng

 

Ta chứng minh bất đẳng thức chặt hơn sau đây

\[\frac{a}{\sqrt{a+b}}+\frac{b}{\sqrt{b+c}}+\frac{c}{\sqrt{c+a}}\leq \frac54\sqrt{a+b+c}.\]

Lời giải.




#567750 Topic tổng hợp một số bất đẳng thức trong kì thi MO các nước

Đã gửi bởi Nguyenhuyen_AG on 23-06-2015 - 23:18 trong Bất đẳng thức - Cực trị

Bài 74:(Poland 2004)

Cho $a,b,c$ là các số thực $a+b+c=0$.CMR:

\[a^2b^2+b^2c^2+c^2a^2+3\geq 6abc. \quad (1)\]

 

Thay $c=-a-b$ vào $(1)$  và đặt $x=a+b,\,y=ab$ ta viết $(1)$ lại dưới dạng

\[y^2+(-2x^2+6x)y+x^4+3 \geqslant 0.\]

Đặt

\[f(y) = y^2+(-2x^2+6x)y+x^4+3,\]

tam thức này có

\[\Delta = (-2x^2+6x)^2-4(x^4+3) = -12(2x+1)(x-1)^2.\]

Do $a+b+c=0$ nên trong ba số $a,\,b,\,c$ sẽ có ít nhất một số âm, giả sử $c \leqslant 0.$ Suy ra $x=a+b = - c \geqslant 0,$ suy ra tiếp

\[\Delta = -12(2x+1)(x-1)^2.\]

Nên theo định lý về dấu của tam thức bậc hai ta có $f(y) \geqslant 0.$ Bài toán được chứng minh.

 

Bài 147 (Làm mạnh bài USAMO 2003). Cho ba số $a,\,b,\,c$ dương. Chứng minh rằng

\[\sum \frac{(2a+b+c)^{2}}{2a^{2}+(b+c)^{2}}+\frac{3(a^2+b^2+c^2)}{(a+b+c)^2} \leqslant 9..\]
 




#567079 Topic tổng hợp một số bất đẳng thức trong kì thi MO các nước

Đã gửi bởi Nguyenhuyen_AG on 20-06-2015 - 14:17 trong Bất đẳng thức - Cực trị

Ở đây có được đăng các bài có trong các tạp chí không vậy, nếu như mỗi MO ko thì nghe vẻ hơi ít

 

Chúng ta chỉ nên đăng những bài trong các kỳ thi MO, TST, ... của các nước, nếu được hãy làm chặt, tổng quát những bài toán đó hoặc nêu một vài dạng tương tự của nó là được. Chẳng hạn như bài toán sau

 

Bài toán 1. (Việt Nam TST 2009) Tìm tất cả các giá trị của $k$ sao cho bất đẳng thức

\[\left ( k+\frac{a}{b+c} \right )\left ( k+\frac{b}{c+a} \right )\left ( k+\frac{c}{a+b} \right ) \geqslant \left ( k+\frac{1}{2} \right )^3,\]

luôn đúng với mọi số thực không âm $a,\,b,\,c$ thỏa mãn $ab+bc+ca > 0.$

(Trần Nam Dũng)

 

 

Có một dạng này có một bài tương tự

 

Bài toán 2. Tìm tất cả các giá trị của $k$ sao cho bất đẳng thức

\[\left ( k+\frac{a}{a+b} \right )\left ( k+\frac{b}{b+c} \right )\left ( k+\frac{c}{c+a} \right ) \geqslant \left ( k+\frac{1}{2} \right )^3.\]

luôn đúng với mọi số thực không âm $a,\,b,\,c$ thỏa mãn $ab+bc+ca > 0.$

 

và một bài tổng quát như sau

 

 

Bài 3. Cho $n \geqslant 3$ số thực không âm $ a_1,a_2,\ldots,a_n$ sao cho $\prod \left (a_1 + a_2 + \cdots + a_{n-1}  \right )\ne 0.$ Chứng minh rằng với với mọi số thực $ k\ge \frac {n - 5 + \sqrt {(n - 1)(n + 7)}}{4(n - 1)}$, ta luôn có
\[ \prod\left ( k + \frac {a_1}{a_2 + a_3 + \cdots + a_n} \right )\ge \left ( k+ \frac 1{n - 1} \right )^n.\]

 




#566295 Topic tổng hợp một số bất đẳng thức trong kì thi MO các nước

Đã gửi bởi Nguyenhuyen_AG on 16-06-2015 - 22:34 trong Bất đẳng thức - Cực trị

Bài 55:(Ailen 2009)

Cho $a,b,c$ là các số thực thỏa $\left\{\begin{matrix} a+b+c=0\\a^2+b^2+c^2=1 \end{matrix}\right.$

Chứng minh rằng

$a^2b^2c^2\leq \frac{1}{54}$

 

Bài này hồi đấy anh chế như vầy. :D

 

Xét bất đẳng thức hiển nhiên sau

\[(a-b)^2(b-c)^2(c-a)^2 \geqslant 0. \quad (1)\]

Đặt $p=a+b+c,\,q=ab+bc+ca,\,r=abc.$ Khi đó khai triển $(1)$ ta được

\[p^2q^2-4q^3+2p(9q-2p^2)r-27r^2 \geqslant 0.\]

Cho $p = a+b+c=0,$ thì

\[-4q^3-27r^2 \geqslant 0,\]

hay

\[4(ab+bc+ca)^3+27a^2b^2c^2 \leqslant 0. \quad (2)\]

Do $a+b+c=0$ nên $a^2+b^2+c^2=(a+b+c)^2-2(ab+bc+ca)=-2(ab+bc+ca),$ do đó bất đẳng thức $(2)$ trở thành

\[(a^2+b^2+c^2)^3 \geqslant 54a^2b^2c^2.\]

Cho $a^2+b^2+c^2=1,$ ta được

\[a^2b^2c^2 \leqslant \frac{1}{54}.\]

Từ đó có bài toán trên. :D




#566290 Topic tổng hợp một số bất đẳng thức trong kì thi MO các nước

Đã gửi bởi Nguyenhuyen_AG on 16-06-2015 - 22:20 trong Bất đẳng thức - Cực trị

Bài 40:
Cho $x,y,z$ là các số thực dương thỏa mãn $x+y+z=xyz$.CMR

$(x-1)(y-1)(z-1)\leq 6\sqrt{3}-10$


Ta chỉ cần xét trường hợp $x-1 > 0,\, y-1 > 0,\, z-1>0.$ Đặt $a = x-1 > 0,\, b = y-1 > 0,\, c = z-1>0,$ khi đó $x=a+1,\,y=b+1,\,z=c+1,$ giả thiết của bài toán trở thành
\[abc+ab+bc+ca=2.\]
Đặt $t=\sqrt[3]{abc},$ áp dụng bất đẳng thức AM-GM ta có
\[2 = abc+ab+bc+ca = t^3+ab+bc+ca \geqslant t^3 + 3\sqrt[3]{{ab}\cdot{bc}\cdot{ca}}=t^3+3t^2.\]
Do $t > 0,$ nên từ bất phương trình $2 \geqslant t^3+3t^2$ ta được $t \leqslant \sqrt{3}-1,$ suy ra $t^3 \leqslant \left(\sqrt{3}-1\right)^3.$ Suy ra tiếp
\[(x-1)(y-1)(z-1) = t^3 \leqslant \left(\sqrt{3}-1\right)^3 = 6\sqrt{3}-10.\]
Bài toán được chứng minh.



#566310 Topic tổng hợp một số bất đẳng thức trong kì thi MO các nước

Đã gửi bởi Nguyenhuyen_AG on 16-06-2015 - 23:18 trong Bất đẳng thức - Cực trị

Bài 80 : Cho $x,y,z$ là các số thực thỏa mãn : $x+y+z=0$ và $x^{2}+y^{2}+z^{2}=6$. Tìm GTLN của : 

$P = \begin{vmatrix} (x-y)(y-z)(z-x) \end{vmatrix}$

 

Từ giả thiết ta suy ra $xy+yz+zx = -3.$ Đặt $p=x+y+z,\,q=xy+yz+zx,\,r=xyz,$ ta có

\[P^2 = (x-y)^2(y-z)^2(z-x)^2= p^2q^2-4q^3+2p(9q-2p^2)r-27r^2.\]

Do $p=0,$ và $q=-3,$ nên

\[P^2 = 54(2-r^2) \leqslant 54 \cdot 2 = 108.\]

Suy ra $P \leqslant 6\sqrt{3}.$ Đẳng thức xảy ra khi

\[ \left\{ \begin{aligned} & xyz = 0 \\& x+y+z=0 \\& {{x}^{2}}+{{y}^{2}}+{{z}^{2}}=6 \\
\end{aligned} \right.  \]

Giải hệ này ta được $(x,y,z) = \left ( 0,\sqrt{3},-\sqrt{3} \right ).$ Vậy $P_{\max} = 6\sqrt{3}.$

 

P/s. ducvipdh12 dạo này bận quá nên anh ít lên diễn đàn. GIờ đang được nghỉ gần 2 tuần nênh giành tí thời gian luyện lại bất đẳng thức, anh thây bên này có vẻ sôi nổi hơn bên Mathscope nhỉ, bên đấy vắng quá.




#566305 Topic tổng hợp một số bất đẳng thức trong kì thi MO các nước

Đã gửi bởi Nguyenhuyen_AG on 16-06-2015 - 22:52 trong Bất đẳng thức - Cực trị

Mình xin nhận xét một vài điều như sau :

III. Bài 75 : Cách giải của nhungviekimcuong rất hay :) . Cách mình cũng không khác cách của bạn ấy nhưng mình xin phân tích bài giải của mình . 

Thực sự , khi gặp các dạng bài như dạng bài 75 , mình cố gằng tìm dấu $=$ nhưng lại không thấy do đó minh chợt nghĩ tới ý tưởng của phương pháp SS với các bđt lệch tâm bằng cách chứng minh một bđt mạnh hơn mà giá trị tại dấu $=$ là tối thiểu . Sau khi thử nhiều số thì nhận thấy với bộ cặp số $(-3,1,1)$ làm cho giá trị $VT-VP$ có vẻ như là nhỏ nhất và đẹp nhất :P . Thế nên không do dự , dựa vào kết quả thừa  mình cộng thêm vế phải một đại lượng $\frac{4}{7}(x+y+z)^{4}$ để bđt xảy ra khi (-3,1,1) và các hoán vị . Nên ta cần chứng minh : $(a+b)^{4}+(b+c)^{4}+(c+a)^{4}\geq \frac{4}{7}(a^{4}+b^{4}+c^{4}+(a+b+c)^{4})$

Tiếp nối ý tưởng của  nhungvienkimcuong  , đặt $\left\{\begin{matrix} a+b=x & & \\ b+c=y & & \\ c+a=z & & \end{matrix}\right.$

Khai triển và rút gọn (Trâu bò đấy :P ) ta được : $24(\sum x^{4})\geq 24\sum (xy)^{2}$ (luôn đúng) (Có thể phân tích sai :) )

 

Bất đẳng thức

\[(a+b)^{4}+(b+c)^{4}+(c+a)^{4}\geq \frac{4}{7}(a^{4}+b^{4}+c^{4}+(a+b+c)^{4}), \quad (2.9.1)\]

được anh Cẩn đề xuất rất lâu trên Mathlinks và được nhắc đến trong quyển Cauchy-Schwarz anh Cẩn. Ta có một chứng minh đơn giản sau đây.

 

Nếu $a+b+c=0,$ thay $c=-a-b$ vào (2.9.1) và thu gọn ta được

\[a^4+2a^3b+3a^2b^2+2ab^3+b^4\ge 0,\]

ta có

\[a^4+2a^3b+3a^2b^2+2ab^3+b^4 = (a^2+ab+b^2)^2 \ge 0.\]

Nếu $a+b+c\ne 0$, thay $(a,\, b,\, c)$ bởi $(-a,\, -b,\, -c)$ thì bất đẳng thức vẫn không thay đổi nên ta có thể giả sử $a+b+c>0,$ và chuẩn hóa cho $a+b+c=3.$

Đặt $a=x+1,\,b=y+1,\,c=z+1,$ thì $x+y+z=0$ và $x^3+y^3+z^3=3xyz,$ ta có

$$(a+b)^4+(b+c)^4+(c+a)^4 = x^4+y^4+z^4+24(x^2+y^2+z^2)-24xyz+48,$$

tương tự thì

$$a^4+b^4+c^4+(a+b+c)^4 = x^4+y^4+z^4+6(x^2+y^2+z^2)+12xyz+84.$$

Như vậy bất đẳng thức trở thành $$x^4+y^4+z^4+48(x^2+y^2+z^2)\ge 72xyz. \quad (2.9.2)$$

Giả sử $xy \ge 0$ rồi thay $z=-x-y$ vào (2.9.2), bất đẳng thức được viết lại dưới dạng

\[2(x^2+xy+y^2)^2+96(x^2+xy+y^2)+72xy(x+y) \ge 0.\]

Theo bất đẳng thức AM-GM, ta có

\[2(x^2+xy+y^2)^2+96(x^2+xy+y^2) \ge 18x^2y^2+72(x+y)^2,\]

lại có

\[x^2y^2+4(x+y)^2+4xy(x+y) = (2x+2y+xy)^2 \ge 0.\]
Đẳng thức xảy ra khi và chỉ khi $a=b=c$ hoặc $-3a=-3b=c.$ Chứng minh của chúng ta vì thế hoàn tất.




#566318 Topic tổng hợp một số bất đẳng thức trong kì thi MO các nước

Đã gửi bởi Nguyenhuyen_AG on 16-06-2015 - 23:57 trong Bất đẳng thức - Cực trị

Bài 109 (IMO 2006). Tìm hằng số $M$ nhỏ nhất sao cho  bất đẳng thức

\[\left | ab(a^2-b^2)+bc(b^2-c^2)+ca(c^2-a^2) \right | \leqslant M(a^2+b^2+c^2)^2,\]

luôn đúng với mọi số thực $a,\,b,\,c$ thay đổi tùy ý.




#566840 Topic tổng hợp một số bất đẳng thức trong kì thi MO các nước

Đã gửi bởi Nguyenhuyen_AG on 19-06-2015 - 13:01 trong Bất đẳng thức - Cực trị

Bài 116 (CĐTMO 2005) : Chứng minh rằng

                                                                 $\frac{a^{3}}{(b+c)^{3}}+\frac{b^{3}}{(c+a)^{3}}+\frac{c^{3}}{(a+b)^{3}}$

                                        trong đó $a,b,c$ là các số dương.

 

Đề bài này sai rồi, đề đúng là như vầy

\[\frac{a^{3}}{(a+b)^{3}}+\frac{b^{3}}{(b+c)^{3}}+\frac{c^{3}}{(c+a)^{3}} \geqslant \frac{3}{8}.\]

Tác giả là thầy Nam Dũng.

 

Lời giải 1. Sử dụng bất đẳng thức Cauchy-Schwarz, ta có \[\left [ \sum \left ( \frac{a}{a+b} \right )^3 \right ]\left [ \sum c^3(a+b)^3 \right ] \ge \left (\sum c^{\frac{3}{2}}a^{\frac{3}{2}} \right )^2,\] như vậy ta cần chứng minh được \[8\left (\sum c^{\frac{3}{2}}a^{\frac{3}{2}}\right )^2 \ge 3 \sum c^3(a+b)^3 .\] Đặt $x=\sqrt{ab},\,y=\sqrt{bc},\,z=\sqrt{ca},$ bất đẳng thức trên trở thành \[8\left (\sum z^3\right )^2 \ge 3 \sum (y^2+z^2)^3,\] hay là \[\sum (x^6+y^6)+16\sum x^3y^3 \ge 9 \sum x^2y^2(x^2+y^2),\] bất đẳng thức này đúng vì \[x^6+y^6+16x^3y^3-9x^2y^2(x^2+y^2) =(x-y)^4(x^2+4xy+y^2) \ge 0.\] Chứng minh hoàn tất.

 

Lời giải 2. Theo bất đẳng thức trung bình lũy thừa, ta có \[\sqrt[3]{\frac{\displaystyle\sum \left ( \frac{a}{a+b} \right )^3}{3}}\ge\sqrt{\frac{\displaystyle\sum \left ( \frac{a}{a+b} \right )^2}{3}}.\] Như vậy, để hoàn tất chứng minh thì ta cần chỉ ra được \[\left ( \frac{a}{a+b} \right )^2+\left ( \frac{b}{b+c} \right )^2+\left ( \frac{c}{c+a} \right )^2 \ge \frac{3}{4}.\] Thay $\left ( \frac{b}{a},\,\frac{c}{b},\,\frac{a}{c} \right )$ bởi $(x,\,y,\,z)$ ta sẽ đưa bài toán về chứng minh \[\frac{1}{(x+1)^2}+\frac{1}{(y+1)^2}+\frac{1}{(z+1)^2}\ge\frac{3}{4},\] với điều kiện $xyz=1.$

 

Sử dụng bất đẳng thức Cauchy-Schwarz, ta có \[\frac{1}{(x+1)^2}+\frac{1}{(y+1)^2}\ge \frac{1}{(\frac{x}{y}+1)(xy+1)}+\frac{1}{(\frac{y}{x}+1)(xy+1)}=\frac{1}{xy+1}.\] Ta sẽ chứng minh \[\frac{1}{xy+1}+\frac{1}{(z+1)^2}\ge \frac{3}{4},\] hay là \[\frac{z}{1+z}+\frac{1}{(z+1)^2}\ge \frac{3}{4}.\] Bất đẳng thức này đúng bởi vì \[\frac{z}{1+z}+\frac{1}{(z+1)^2}-\frac{3}{4}=\frac{(z-1)^2}{4(z+1)} \ge 0.\]
Nhận xét. Một cách khác để chứng minh bất đẳng thức \[\left ( \frac{a}{a+b} \right )^2+\left ( \frac{b}{b+c} \right )^2+\left ( \frac{c}{c+a} \right )^2 \ge \frac{3}{4}.\] Sử dụng bất đẳng thức Cauchy-Schwarz, ta có \[\left [ \sum \left ( \frac{a}{a+b} \right )^2 \right ]\left [ \sum (a+b)^2(a+c)^2 \right ]\ge \left [ \sum a(a+c) \right ]^2.\] Tuy nhiên \[4\left [ \sum a(a+c) \right ]^2 = \left [ \sum (a+b)^2 \right ]^2\] Vậy, ta chỉ cần chứng minh \[\left [ \sum (a+b)^2 \right ]^2 \ge 3 \sum (a+b)^2(a+c)^2,\] nhưng bất đẳng thức này đúng theo bất đẳng thức cơ bản \[(x+y+z)^2\ge 3(xy+yz+zx).\] Bất đẳng thức  được chứng minh.

 

Ngoài ra ta có thể chứng minh bài toán bằng cách sử dụng bất đẳng thức China TST 2004 của giáo sư Vasile Cirtoaje

\[\frac{a^2}{(a+b)^2}+\frac{b^2}{(b+c)^2}+\frac{c^2}{(c+d)^2}+\frac{d^2}{(d+a)^2} \geqslant 1.\]

 

 

Bài 115 (CĐTMO 2006) : Chứng minh rằng với mọi số thực $x,y,z$ thuộc đoạn $[1;2]$ ta luôn có bất đẳng thức :
                                          $(x+y+z)(\frac{1}{x}+\frac{1}{y}+\frac{1}{z})\geq 6(\frac{x}{y+z}+\frac{y}{x+z}+\frac{z}{x+y}).$

                                           Hỏi đẳng thức xảy ra khi và chỉ khi nào ?
 

 

Đặt \[f(x,\,y,\,z)=(x+y+z)\left( \frac{1}{x}+\frac{1}{y}+\frac{1}{z} \right)-6\left( \frac{x}{y+z}+\frac{y}{z+x}+\frac{z}{x+y} \right),\]

ta sẽ chứng minh $f(x,\,y,\,z)\ge 0$ bằng kỹ thuật dồn biến. Thật vậy, ta có $f(x,\,y,\,z)-f\left ( x,\,\frac{y+z}{2},\, \frac{y+z}{2}\right ),$ sẽ bằng \[\frac{(x+y+z)(y-z)^2}{yz(y+z)}-\frac{6(x+y+z)(y-z)^2}{(x+y)(x+z)(2x+y+z)},\] hay \[\frac{(x+y+z)(y-z)^2[(x+y)(z+x)(2x+y+z)-6yz(y+z)]}{(x+y)(y+z)(z+x)(2x+y+z)}.\] Giả sử $x$ là số lớn nhất trong ba số thì $(x+y)(z+x)(2x+y+z) > 6yz(y+z),$ nên $f(x,\,y,\,z)\ge f(x,\,t,\,t)$ với $t=\frac{y+z}{2} \ge 1.$

 

Tiếp đến, ta sẽ chứng minh $f(x,\,t,\,t)\ge 0.$ Bất đẳng thức tương đương với

$$(x+2t)\left( \frac{1}{x}+\frac{2}{t} \right)-6\left( \frac{x}{2t}+\frac{2t}{t+x} \right)\ge 0,$$

$$\frac{{{(t-x)}^{2}}(2t-x)}{tx(t+x)}\ge 0,$$

hiển nhiên đúng vì $2t\ge 2\ge x.$ Đẳng thức xảy ra khi và chỉ khi $x=y=z,$ hoặc $x=2$ và $y=z=1.$ Bài toán được chứng minh.

 

Nhận xét. Nếu $x,\,y,\,z$ là độ dài ba cạnh của tam giác thì bài toán vẫn đúng. Thật vậy, vì $x,\,y,\,z$ là độ dài ba cạnh của tam giác nên, theo phép thế Ravi, ta có thể viết bất đẳng thức lại như sau \[(x+y+z)\sum \frac{1}{x+y} \ge 3\sum \frac{y+z}{2x+y+z},\] tương đương với \[2\sum x^2(x^2-y^2)(x^2-z^2)+3\sum yz(y^2-z^2)^2 \ge 0,\] hiển nhiên đúng theo bất đẳng thức Shur.

 

Tác giả bài toán cũng là thầy Nam Dũng.




#567076 Topic tổng hợp một số bất đẳng thức trong kì thi MO các nước

Đã gửi bởi Nguyenhuyen_AG on 20-06-2015 - 14:06 trong Bất đẳng thức - Cực trị

Bài 117 (CĐTMO 2001) : Xét các số thực dương $a,b,c$ thỏa mãn điều kiện 

                                                                          $21ab+2bc+8ca\leq 12$
                                        Tìm giá trị nhỏ nhất của biểu thức

\[P=\frac{1}{a}+\frac{2}{b}+\frac{3}{c}.\]

 

Tác giả bài này là thầy Nam Dũng.

 

Lời giải 1. (Có thể đây là đáp chính thức) Đặt $x=\frac{1}{a},\,y=\frac{2}{b},\,z=\frac{3}{c}$ ta có thể viết giả thiết của của bài toán lại thành $2xyz\ge 2x+4y+7z$ và cần tìm giá trị nhỏ nhất của

$$P = x+y+z.$$
Giả thiết được viết lại dưới dạng $z(2xy-7)\ge 2x+4y,$ từ đó suy ra $2xy>7,\,\,z\ge \frac{2x+4y}{2xy-7}.$ Với điều kiện này, áp dụng bất đẳng thức AM-GM, ta được \[\begin{aligned} f(x,\,y,\,z)=x+y+z&\ge x+y+\frac{2x+4y}{2xy-7}\\ &= x+\frac{11}{2x}+\left (y-\frac{7}{2x} \right )+\left (\frac{2x+4y}{2xy-7}-\frac{2}{x} \right )\\&= x+\frac{11}{2x}+\frac{2xy-7}{2x}+\frac{2(x^2+7)}{x(2xy-7)}\\&\ge x+\frac{11}{2x}+2\sqrt{\frac{2xy-7}{2x}\cdot\frac{2(x^2+7)}{x(2xy-7)}}\\& =x+\frac{11}{2x}+2\sqrt{1+\frac {7}{x^{2}}}.\end{aligned}\] Mặt khác theo bất đẳng thức Cauchy-Schwarz thì

$$(9+7)\left(1+\frac{7}{x^2}\right)\ge\left( 3+\frac{7}{x}\right)^2.$$

Từ đó theo bất đẳng thức AM-GM, ta được

$$x+\frac{11}{2x}+2\sqrt{1+\frac{7}{x^2}}\ge x+\frac{11}{2x}+\frac{1}{2}\left(3+\frac{7}{x}\right)=\left(x+\frac{9}{x}\right)+\frac{3}{2}\ge\frac{15}{2}.$$

Đẳng thức xảy ra khi và chỉ khi $x=3,\,y=\frac{5}{2},\,z=2.$ Như vậy, giá trị nhỏ nhất cần tìm là $\frac{15}{2}.$

 

 

Lời giải 2. Tương tự như trên ta cũng thực hiện phép đổi biến để đưa bài toán về tìm giá trị nhỏ nhất của biếu thức

$$P = x+y+z,$$

với điều kiện $2xyz \ge 2x+4y+7z.$

 

Sử dụng bất đẳng thức AM-GM suy rộng, ta có

$$x+y+z=\frac{5x}{2}\cdot \frac{2}{5}+3y\cdot \frac{1}{3}+\frac{15z}{4}\cdot \frac{4}{15}\ge {{\left( \frac{5x}{2} \right)}^{\frac{2}{5}}}{{\left( 3y \right)}^{\frac{1}{3}}}{{\left( \frac{15z}{4} \right)}^{\frac{4}{15}}}.$$

Hoàn toàn tương tự

$$2x+4y+7z\ge {{10}^{\frac{1}{5}}}\cdot {{12}^{\frac{1}{3}}}\cdot {{5}^{\frac{7}{15}}}\cdot {{x}^{\frac{1}{5}}}\cdot {{y}^{\frac{1}{3}}}\cdot {{z}^{\frac{7}{15}}}.$$

Nhân tương ứng hai bất đẳng thức trên lại với nhau, ta được

$${{(x+y+z)}^{2}}(2x+4y+7z)\ge \frac{225}{2}xyz.$$

Tuy nhiên vì $2xyz\ge 2x+4y+7z,$ nên

$${{(x+y+z)}^{2}}(2x+4y+7z)\ge \frac{225}{4}(2x+4y+7z),$$ tức là

$$x+y+z\ge \frac{15}{2}.$$

Từ đó suy ra kết quả như trên.

Nhận xét. Sử dụng bất đẳng thức AM-GM, ta thấy rằng

$$\begin{aligned} x&=\frac{4}{5b}+\frac{3}{2c}+\frac{5bc}{6}-3 \ge 0, \\ y&=\frac{3}{2c}+\frac{1}{3a}+2ca-3 \ge 0, \\z&=\frac{1}{3a}+\frac{4}{5b}+\frac{15ab}{4}-3 \ge 0.\end{aligned}$$

Khi đó, bằng cách sử dụng phân tích

$$f(a,\,b,\,c)=\frac{15}{2}+\frac{5}{6}(12-21ab-2bc-8ca)+\frac{3x+5y+7z}{4},$$

ta dễ dàng suy ra điều phải chứng minh.

 

Với bất đẳng thức AM-GM suy rộng, ta có thể giải quyết được bài toán tổng quát của bất đẳng thức trên một bài toán từng xuất hiện trong đề thi VMEO của VMF.
 

Với $x,\,y,\,z$ là các số dương thay đổi và $a,\,b,\,c$ là các số dương cho trước thỏa mãn điều kiện $ax+by+cz=xyz.$ Khi đó ta có bất đẳng thức sau đây

$$x+y+z \ge \frac{2}{d}\sqrt{(a+d)(b+d)(c+d)},$$

trong đó $d$ là nghiệm dương của phương trình

$$\frac{1}{a+d}+\frac{1}{b+d}+\frac{1}{c+d}=\frac{2}{d}.$$

 




#566423 Topic tổng hợp một số bất đẳng thức trong kì thi MO các nước

Đã gửi bởi Nguyenhuyen_AG on 17-06-2015 - 13:53 trong Bất đẳng thức - Cực trị

Bài 55:(Ailen 2009)

Cho $a,b,c$ là các số thực thỏa $\left\{\begin{matrix} a+b+c=0\\a^2+b^2+c^2=1 \end{matrix}\right.$

Chứng minh rằng

$a^2b^2c^2\leq \frac{1}{54}$

 

 

Ngoài lời giải theo con đường khai triển $(a-b)^2(b-c)^2(c-a)^2 \geqslant 0,$ anh vừa tìm được một chứng minh tự nhiên hơn (lúc sáng đang xem Argentina đá chợt nghĩ ra :D)

 

Ta thấy bài toán là hệ quả của bất đẳng thức sau

\[(a^2+b^2+c^2)^3 \geqslant  54a^2b^2c^2, \quad (2)\]

Thật vậy theo bất đẳng thức Cauchy-Schwarz ta được

\[a^2+b^2+c^2 \geqslant \frac{(a+b)^2}{2}+c^2=\frac{3c^2}{2},\]

suy ra

\[(a^2+b^2+c^2)^3 \geqslant \frac{27c^6}{8}.\]

Như vậy ta chỉ cần chứng minh

\[\frac{27c^6}{8} \geqslant 54a^2b^2c^2,\]

hay

\[c^2 \geqslant 4\left | ab \right |.\]

Áp dụng bất đẳng thức AM-GM, ta có

\[4\left | ab \right | \leqslant \left |a+b  \right |^2 =(a+b)^2 = c^2.\]

Bài toán được chứng minh.

 

Nhận xét. Nếu thay $c=-a-b$ vào $(2)$ ta được

\[[a^2+b^2+(a+b)^2]^3 \geqslant  54a^2b^2(a+b)^2.\]

Ta có

\[[a^2+b^2+(a+b)^2]^3 - 54a^2b^2(a+b)^2=2(a-b)^2(2a+b)^2(2b+a)^2 \geqslant 0.\]

Đây cũng là một lời giải cho bài toán.

 




#588629 Bất đẳng thức chuẩn bị cho kì thi THPTQG 2015-2016

Đã gửi bởi Nguyenhuyen_AG on 13-09-2015 - 00:21 trong Bất đẳng thức và cực trị

Cho a,b,c là các số không âm trong đó không có 2 số nào đồng thời bằng 0. Chứng minh rằng: $\sum \frac{3a^{3}+abc}{b^{3}+c^{3}}\geq 6$

Thay $(a,\,b,\,c)$ bởi $(\sqrt[3]{a},\,\sqrt[3]{b},\,\sqrt[3]{c})$ ta cần chứng minh \[\frac{3a+\sqrt[3]{abc}}{b+c}+\frac{3b+\sqrt[3]{abc}}{c+a}+\frac{3c+\sqrt[3]{abc}}{a+b}\geq 6.\] Áp dụng bất đẳng thức Cauchy-Schwarz, ta có \[\sum \frac{a}{b+c} \geqslant \frac{(a+b+c)^2}{2(ab+bc+ca)} = \frac{a^2+b^2+c^2}{ab+bc+ca} + 2,\] và \[\sum \frac{1}{b+c} \geqslant \frac{9}{2(a+b+c)}.\] Như vậy ta chỉ cần chỉ ra \[\frac{a^2+b^2+c^2}{ab+bc+ca}+\frac{3\sqrt[3]{abc}}{a+b+c} \geqslant 2,\] hay \[\frac{3\sqrt[3]{abc}(ab+bc+ca)}{a+b+c} \geqslant 2(ab+bc+ca)-a^2-b^2-c^2.\] Theo bất đẳng thức Schur bậc ba thì \[\frac{9abc}{a+b+c} \geqslant 2(ab+bc+ca)-a^2-b^2-c^2.\] Do đó bất đẳng thức sẽ được chứng minh nếu \[ab+bc+ca \geqslant 3\sqrt[3]{a^2b^2c^2}.\] Hiển nhiên đúng theo bất đẳng thức AM-GM.

Nhận xét. Vì \[\frac{3\sqrt[3]{abc}}{a+b+c} \geqslant \frac{8abc}{(a+b)(b+c)(c+a)}\] nên ta có bất đẳng thức mạnh hơn \[\frac{a^2+b^2+c^2}{ab+bc+ca} + \frac{8abc}{(a+b)(b+c)(c+a)} \geqslant 2.\] Đây là một kết quả quen thuộc của Jack Garfunkel.



#679401 Topic BẤT ĐẲNG THỨC ôn thi vào lớp 10 THPT 2017 - 2018

Đã gửi bởi Nguyenhuyen_AG on 03-05-2017 - 21:49 trong Bất đẳng thức và cực trị

tại sao anh tìm đc đánh giá trên vậy?

 

Anh dùng hệ số bất định kết hợp với đạo hàm.

 

Có cách giải nào ngoài phương pháp Đ giá đại diện sau khi dùng đạo hàm như anh k ạ?

Cách này em biết rồi nhưng ở đây ta cần một lời giải phù hợp THCS hơn ạ. 

Ps: chỉ là góp ý nho nhỏ,

 

Đổi biến $(a,b,c) \to \left(\frac{bc}{a^2},\frac{ca}{b^2},\frac{ab}{c^2}\right)$ và đánh giá bằng bất đẳng thức Holder

\[\left(\sum \frac{a^2}{\sqrt{a^4+6a^2bc+2b^2c^2}}\right)^2 \sum a^2(a^4+6a^2bc+2b^2c^2) \geqslant (a^2+b^2+c^2)^3.\]




#680649 Topic BẤT ĐẲNG THỨC ôn thi vào lớp 10 THPT 2017 - 2018

Đã gửi bởi Nguyenhuyen_AG on 14-05-2017 - 14:20 trong Bất đẳng thức và cực trị

Bài toán 61(sưu tầm)

Cho a,b,c là các số thực không âm, đôi một khác nhau. CMR:

$(ab+bc+ca)\left [ \frac{1}{(a-b)^{2}}+\frac{1}{(b-c)^{2}}+\frac{1}{(c-a)^{2}} \right ]\geqslant 4$

 

Ta có

\[\text{Vế trái  -Vế phải} = \sum \frac{ab\left[c(a+b-c)+a^2-3ab+b^2\right]^2}{(a-b)^2(b-c)^2(c-a)^2} \geqslant 0.\]

P/s. Anh nhớ Doflamingo bị Hải quân bắt rồi sao còn ở đây post bài nhỉ?




#678809 Topic BẤT ĐẲNG THỨC ôn thi vào lớp 10 THPT 2017 - 2018

Đã gửi bởi Nguyenhuyen_AG on 27-04-2017 - 23:01 trong Bất đẳng thức và cực trị

Làm trội lên :

(Lê Khánh Sỹ, tạp chí toán học rumania)

Cho $a, b, c \geq 0$; $c=min\left \{ a,b,c \right \}$ . Chứng minh rằng:

$$\frac{a^{2}+b^{2}+c^{2}}{ab+bc+ca}+ \frac{8abc}{(a+b)(b+c)(c+a)} \geq 2 + \frac{2c(a-b)^{2}}{3(a+b)(b+c)(c+a)}$$

 

Ta có

\[\text{VT-VP} = \frac{\left[(ab+2bc+5ca)(a-c)+(2a^2+3ab+3bc)(b-c)\right](a-b)^2+3c^2(a+b)(a-c)(b-c)}{3(a+b)(b+c)(c+a)(ab+bc+ca)} \geqslant 0.\]




#677981 Topic BẤT ĐẲNG THỨC ôn thi vào lớp 10 THPT 2017 - 2018

Đã gửi bởi Nguyenhuyen_AG on 19-04-2017 - 00:27 trong Bất đẳng thức và cực trị

$\boxed{8}$ [Trần Quốc Anh] Cho $a,b,c$ là các số thực không âm thỏa mãn $a+b+c=2$. Chứng minh rằng:

\[(a^2+ab+b^2)(b^2+bc+c^2)(c^2+ca+a^2) \le 3 \]

 

Bất đẳng thức cần chứng minh tương đương với

\[\frac{3}{64}(a+b+c)^6 \geqslant (a^2+ab+b^2)(b^2+bc+c^2)(c^2+ca+a^2),\]

hoặc

\[\frac{5}{16}(a+b+c)^2 \sum ab(a-b)^2+\frac{1}{64}\left(\sum a^2-2\sum bc\right)^2\left(3\sum a^2 + 10 \sum bc \right) + \frac{1}{16}abc(a+b+c)^3 \geqslant 0.\]

Từ đó suy điều phải chứng minh.

 

P/s. Bài này không phải của Trần Quốc Anh đâu.




#679393 Topic BẤT ĐẲNG THỨC ôn thi vào lớp 10 THPT 2017 - 2018

Đã gửi bởi Nguyenhuyen_AG on 03-05-2017 - 21:28 trong Bất đẳng thức và cực trị

BÀI TOÁN 41( Sưu tầm):  Cho $a,b,c\ge0$ thỏa mãn $abc=1$

CMR: $\frac{1}{\sqrt{2a^2+6a+1}}+\frac{1}{\sqrt{2b^2+6b+1}}+\frac{1}{\sqrt{2c^2+6c+1}}\geq1$

 

Chỉ cần chứng minh

\[\frac{1}{\sqrt{2a^2+6a+1}} \geqslant \frac{1}{a^{10/9} + a^{5/9} + 1}.\]